LSAT and Law School Admissions Forum

Get expert LSAT preparation and law school admissions advice from PowerScore Test Preparation.

 Administrator
PowerScore Staff
  • PowerScore Staff
  • Posts: 8919
  • Joined: Feb 02, 2011
|
#26785
(The complete setup for this game can be found here: lsat/viewtopic.php?t=11235)

Please post below with any questions!
 Tw07
  • Posts: 7
  • Joined: Feb 15, 2020
|
#73872
I used the inferences from my set up to eliminate some, and then had to spend some time with the choices I had left diagramming. How would you approach this question quickly?
 Jeremy Press
PowerScore Staff
  • PowerScore Staff
  • Posts: 1000
  • Joined: Jun 12, 2017
|
#73900
Hi Tw07,

On these Justify questions (questions where you're asked to find an answer that would definitively force a certain result in the game), it's best to try to "predict" a good kind of answer to test.

We want to find an answer choice that would force Mollie into 1922.

1. We know Mollie can only ever occupy 1921 or 1922. So, if an answer choice placed a different variable into 1921, that would be a good starting point. Why? Because with some other variable in 1921, it becomes somewhat more likely Mollie will have to go in 1922.

2. That draws my attention to answer choices B and E. But B is not a good answer choice, because I know from my setup that there are certain situations where, when O is in 1921, R (not M) is in 1922. For example, as I was setting up the game, I thought about what might happen if the last two conditional rules "triggered", and I got these templates:
Screen Shot 2020-02-17 at 1.28.31 PM.png
3. So I test answer choice E. Placing Y into 1921 means both T and R must be out (if either one of them were in, then I'd have to fill 1921 and 1922 with an OR block). That means M must be in, and the only spot she can then occupy is 1922.

I hope this helps!

Jeremy
 hhaugen@ualberta.ca
  • Posts: 4
  • Joined: Jun 22, 2021
|
#88569
Hello,

I am struggling to understand why O assigned to 1921 does not have the same effect as Y being assigned to 1921? Why is E correct but not B?

Thanks!

Hannah
 Rachael Wilkenfeld
PowerScore Staff
  • PowerScore Staff
  • Posts: 1358
  • Joined: Dec 15, 2011
|
#88588
Hi Hannah,

The key to placing Y in 1921 is that it breaks up the years so that you cannot have an OR block. If you can't have an OR block, that means you can't have R. And if you can't have R, you can't have T. That fills up our out group with R and T, meaning everyone else, including M has to be used. M can only be in 1921 or 1922, and Y is in 1921 here. That forces M into 1922.

If we assign O to 1921, we can still have our OR block in 1921/1922, and M could be left out. O in 21 doesn't force M in 22 because we have other options.

Hope that helps!
User avatar
 shasmcg
  • Posts: 1
  • Joined: Oct 27, 2022
|
#98856
I am going through this question and I'm a bit puzzled as the as the last two rules do not seem to preclude O from appearing if R and T are not selected.
Can you explain?
 Adam Tyson
PowerScore Staff
  • PowerScore Staff
  • Posts: 5153
  • Joined: Apr 14, 2011
|
#98866
T requires R, and R requires O, shasmcg, but O does not require either R or T. That would be a Mistaken Reversal of those conditional rules.

Here's an analogy that might help:

To travel to a foreign country, you need a passport, and to get a passport, you need a photograph of yourself.

Now, can I have a photograph of myself without having a passport or going to a foreign country? Of course I can! My photo doesn't require those things, even though those things require my photo.

So, having R and T out, with LMOY in, works just fine. L would go 3rd, per the first rule. M would not go 4th, so that would have to be either O or else Y. M would go 1st or 2nd, and either Y or O would fill the remaining spot (2nd or 1st). This breaks none of the rules! Those last two rules only apply IF T or R is assigned, and since they are not assigned, those rules are never activated.
User avatar
 marissasalazar9899
  • Posts: 5
  • Joined: Aug 25, 2023
|
#104736
Hello, I am confused as to why the answer is not C. I was stuck between C and E because as I was solving the problem my solution was that y is in '21, m is '22 and L is in '23 and O is in '24. Why can't O be in '24?
 Adam Tyson
PowerScore Staff
  • PowerScore Staff
  • Posts: 5153
  • Joined: Apr 14, 2011
|
#104760
Because in that scenario, marissasalazar9899 , M and Y could switch places. M COULD be in 1922, but M does not have to be. Instead, the solution could me MYLO.

The question isn't about where O could be, but about which answer would force M to be in 1922. If Y is in 1921, R and T are out, and since M cannot be in 1924, the only place left for it to go is in 1922 (which also means L is in 1923 and O is in 1924.)

Get the most out of your LSAT Prep Plus subscription.

Analyze and track your performance with our Testing and Analytics Package.